LSAT and Law School Admissions Forum

Get expert LSAT preparation and law school admissions advice from PowerScore Test Preparation.

 Administrator
PowerScore Staff
  • PowerScore Staff
  • Posts: 8923
  • Joined: Feb 02, 2011
|
#23941
Complete Question Explanation

Main Point. The correct answer choice is (C)

Contrary to the claims made in the newspaper article on Britain’s unions, the author concludes that the strength of these unions is not declining. The author’s rationale is that strong unions do not need to call strikes, which is why the decreasing number of strikes is evidence of strength and not weakness:
  • Premise (1) ..... The number of strikes in Britain is falling.

    Premise (2) ..... Strong unions do not need to call strikes.

    Sub Conclusion: ..... The calling of a strike is evidence that the union is too weak.

    Main Conclusion: ..... The strength of Britain’s unions is not declining (i.e. the article is mistaken in concluding that it is).
The main point of the argument is that the newspaper article is wrong. This conclusion is implicit in the author’s argument, and should be prephrased before examining the five answer choices. Remember: whenever the author begins by describing someone else’s viewpoint, you should anticipate that her position will be the direct opposite of theirs. Answer choice (C) best captures the main point of the argument and is therefore correct.

Answer choice (A): This is the central premise of the argument, not its main conclusion. Indeed, the author does suggest that strikes are evidence of weakness on the part of the unions, because strong unions do not need to call strikes. However, this observation is further used to support the conclusion that the newspaper article is wrong in suggesting that Britain’s unions are weak.

Answer choice (B): The information contained in the stimulus provides no support for this conclusion. This answer choice is incorrect.

Answer choice (C): This is the correct answer choice. See discussion above.

Answer choice (D): This answer choice can only be a contender if you assume that the conclusion of any argument is bound to appear at the end of the stimulus. This is rarely the case in Main Point questions. The last sentence in this case merely clarifies a premise used in support of the main conclusion of the argument.

Answer choice (E): The information contained in the stimulus provides no support for this conclusion. This answer choice is incorrect.
 voodoochild
  • Posts: 185
  • Joined: Apr 25, 2012
|
#5588
Instructors,
I guess that A is a restatement of "Surely, in a modern industrial society, the calling of strike is an evidence that the negotiating position of the union is too weak."

A actually uses an extreme "the only". I believe that A) is incorrect because it's not the main conclusion. It's an intermediate conclusion. It's not incorrect because of the extreme phrase "the only". In my opinion, the logical diagram for the above intermediate conclusion and A) is the same...

I just want to make sure that I am eliminating the answer choice for the right reason!

Am I correct?
 Steve Stein
PowerScore Staff
  • PowerScore Staff
  • Posts: 1153
  • Joined: Apr 11, 2011
|
#5632
Hey Voodoo,

I don't think this one requires any diagramming. Answer choice A is a premise that is presented in the stimulus, so it cannot be the author's conclusion; the author's main goal is to take issue with the newspaper argument presented in the first two sentences.

I hope that's helpful! Let me know--thanks!

~Steve

Get the most out of your LSAT Prep Plus subscription.

Analyze and track your performance with our Testing and Analytics Package.